write down in trems of n an expression for the nth term in the following sequences 1 8 15 22 29

Answers

Answer 1

The expression which defines the nth term of the given sequence, in terms of n as required in the task content is; T (n) = 1 + 7(n - 1).

Which expression represents the nth term of the expression in terms of n?

It follows from the task content that the expression which represents the nth term of the given sequence be determined.

By observation, the first term, a of the sequence is; 1.

Also, since; 8 - 1 = 15 - 8 = 22 - 15 = 29 - 22 = Common difference, d = 7.

It follows that the expression is a arithmetic sequence which can be modelled by;

T (n) = a + d (n - 1)

Since the constant common difference of consecutive terms in the sequence is; 7,

The required expression for the nth term of the expression is;

nth = 1 + 7(n -1)

On this note, the nth term is given by the expression; T (n) = 1 + 7 (n -1).

Read more on arithmetic sequence;

https://brainly.com/question/22972770

#SPJ1


Related Questions

Rewrite as equivalent rational expressions with denominator (3m−4)(m+8)(m−7):
63m2+20m−32,3m3m2−25m+28

Answers

The first rational expression is 63m²+20m−32. To rewrite this with a common denominator of (3m−4)(m+8)(m−7), first we need to find the LCD (Lowest Common Denominator) of both terms. So, we need to multiply each numerator and denominator of the first rational expression by (3m−4)(m+8)(m−7).

In the numerator, we need to multiply 63m² by (3m−4)(m+8)(m−7):

63m² × (3m−4)(m+8)(m−7) = 63m³ - 224m² + 784m - 504

In the denominator, we need to multiply 1 by (3m−4)(m+8)(m−7):

1 × (3m−4)(m+8)(m−7) = 3m³ - 12m² + 24m - 16

Therefore, the rewritten rational expression is:

63m³ - 224m² + 784m - 504/3m³ - 12m² + 24m - 16

The second rational expression is 3m³m²−25m+28. To rewrite this with a common denominator of (3m−4)(m+8)(m−7), we first need to find

In KLM, KM is extended through point M to point N,
(3x +19)°, m/LMN = (7x + 5)°, and
m/KLM = (2x+8)°. What is the value of x?

Answers

The value of the variable "x" is 11.

We have a triangle. The vertices of the triangle are K, L, and M. The side KM is extended from point M to point N. The measures of the angles MKL, LMN, and KLM are (3x + 19)°, (7x + 5)°, and (2x + 8)°, respectively. We need to find out the value of the variable "x".

The angles LMK and LMN form a linear pair. It means they are supplementary angles. The sum of the angles is 180°.

∠LMK + ∠LMN = 180°

∠LMK + (7x + 5)° = 180°

∠LMK = 180° - (7x + 5)°

In the triangle KLM, we will use the angle sum property of a triangle. The sum of all the angles in a triangle is equal to 180°.

∠K + ∠L + ∠M = 180°

(3x +19)° + (2x + 8)° + [180° - (7x + 5)°] = 180°

3x +19 + 2x + 8 + 180 - 7x - 5 = 180

-2x + 22 = 0

2x = 22

x = 11

Hence, the value of the variable "x" is 11.

To learn more about variables, visit :

https://brainly.com/question/22277991

#SPJ9

Can someone PLEASE answer 7 and 8 this assignment is due in a few minutes

Can someone PLEASE answer 7 and 8 this assignment is due in a few minutes

Answers

Step-by-step explanation:

7

not equivalent

8

6.11 $

A square has sides that measure 8x + 5 inches. A rectangle has two sides that measure 4x - 5 inches and two sides that measure 12x + 15 inches.

For what value of x will the perimeters of the square and rectangle be the same?​

Answers

Answer:

The perimeter of the square is 32x + 20 inches, and the perimeter of the rectangle is (4x - 5) + (4x - 5) + (12x + 15) + (12x + 15) = 32x + 20 inches.

So, to find the value of x that makes the perimeters the same, we set 32x + 20 = 32x + 20 and solve for x.

x = 0

Step-by-step explanation:

Hurry plz will give u brainliest

Hurry plz will give u brainliest

Answers

Answer:

3

Step-by-step explanation:

Please explain how to do it too ill give brainliest

Please explain how to do it too ill give brainliest

Answers

Answer:

x = 90

Step-by-step explanation:

The given diagram shows a circle with intersecting chords, KM and JL.

To find the value of x, we can use the Angles of Intersecting Chords Theorem.

According to the Angles of Intersecting Chords Theorem, if two chords intersect within a circle, the angle formed at the intersection point is equal to half the sum of the measures of the arcs intercepted by the angle and its corresponding vertical angle.

Let the point of intersection of chords KM and JL be point P.

As the chords are straight lines, angle x° forms a linear pair with angle JPM.

Note: We cannot use the Angles of Intersecting Chords Theorem to find the value of x directly, since we have not been given the measures of the arcs KJ and ML. Therefore, we need to use the theorem to find m∠JPM first.

From inspection of the given diagram:

\(m\overset\frown{JM}=30^{\circ}\)\(m\overset\frown{LK}=(2x - 30)^{\circ}\)

Using the Angles of Intersecting Chords Theorem, we can calculate the measure of angle JPM (shown in orange on the attached diagram):

\(\begin{aligned}m \angle JPM &=\dfrac{1}{2}\left(m\overset\frown{JM}+m\overset\frown{LK}\right)\\\\&=\dfrac{1}{2}\left(30^{\circ}+(2x-30)^{\circ}\right)\\\\&=\dfrac{1}{2}\left(30^{\circ}+2x^{\circ}-30^{\circ}\right)\\\\&=\dfrac{1}{2}\left(2x^{\circ}\right)\\\\&=x^{\circ}\end{aligned}\)

As angle JPM forms a linear pair with angle x°, the sum of the two angles equals 180°:

\(\begin{aligned}m \angle JPM+x^{\circ}&=180^{\circ}\\\\x^{\circ}+x^{\circ}&=180^{\circ}\\\\2x^{\circ}&=180^{\circ}\\\\\dfrac{2x^{\circ}}{2}&=\dfrac{180^{\circ}}{2}\\\\x^{\circ}&=90^{\circ}\\\\x&=90\end{aligned}\)

Therefore, the value of x is 90, which means that the two chords intersect at right angles.

Please explain how to do it too ill give brainliest

HELP ASAP 50 Points ASAP

HELP ASAP 50 Points ASAP

Answers

Answer:

a=0.2 b=86

Step-by-step explanation:

in this equation, a is the coefficient (what the base is multiplied by) and b is the base (what you start out with).  because isabella started off with $86 so that is her base and because her sales increased by 20% so that is her coefficient

can anyone explain this thank you

can anyone explain this thank you

Answers

Answer:

89.8ft

Step-by-step explanation:

678-672=6, the value for line ZW

Using Pythagoras theorem, then t²+6²=90²

t²=90²-6²

t²=8100-36

√t²=√8064

t=89.8ft

Angle 6 and angle 7 are related by

Answers

Answer:the answer is 5 and 8 i guess

Step-by-step explanation: For example, angles 1 and 2 are both facing in the same direction, to the upper right. Such angles are called corresponding angles. Similarly we have angles 3 and 6, angles 4 and 7, and angles 8 and 5 as corresponding angles.

the area of a triangle is 16cm^2, if the base of the triangle is two less than its height. find the base and the height​

Answers

Answer:

The base of the triangle is 4cm and the height is 6cm

PRE CALC HELP NEEDED

PRE CALC HELP NEEDED

Answers

Answer:

\(\dfrac{5e^2}{2}\)

Step-by-step explanation:

Differentiation is an algebraic process that finds the slope of a curve. At a point, the slope of a curve is the same as the slope of the tangent line to the curve at that point. Therefore, to find the slope of the line tangent to the given function, differentiate the given function.

Given function:

\(y=x^2\ln(2x)\)

Differentiate the given function using the product rule.

\(\boxed{\begin{minipage}{5.5 cm}\underline{Product Rule for Differentiation}\\\\If $y=uv$ then:\\\\$\dfrac{\text{d}y}{\text{d}x}=u\dfrac{\text{d}v}{\text{d}x}+v\dfrac{\text{d}u}{\text{d}x}$\\\end{minipage}}\)

\(\textsf{Let\;$u=x^2}\)\(\textsf{Let\;$u=x^2$}\implies \dfrac{\text{d}u}{\text{d}x}=2x\)

\(\textsf{Let\;$v=\ln(2x)$}\implies \dfrac{\text{d}v}{\text{d}x}=\dfrac{2}{2x}=\dfrac{1}{x}\)

Input the values into the product rule to differentiate the function:

\(\begin{aligned}\dfrac{\text{d}y}{\text{d}x}&=u\dfrac{\text{d}v}{\text{d}x}+v\dfrac{\text{d}u}{\text{d}x}\\\\&=x^2 \cdot \dfrac{1}{x}+\ln(2x) \cdot 2x\\\\&=x+2x\ln(2x)\end{aligned}\)

To find the slope of the tangent line at x = e²/2, substitute x = e²/2 into the differentiated function:

\(\begin{aligned}x=\dfrac{e^2}{2}\implies \dfrac{\text{d}y}{\text{d}x}&=\dfrac{e^2}{2}+2\left(\dfrac{e^2}{2}\right)\ln\left(2 \cdot \dfrac{e^2}{2}\right)\\\\&=\dfrac{e^2}{2}+e^2\ln\left(e^2\right)\\\\&=\dfrac{e^2}{2}+2e^2\\\\&=\dfrac{5e^2}{2}\end{aligned}\)

Therefore, the slope of the line tangent to the graph of y = x²ln(2x) at the point where x = e²/2 is:

\(\boxed{\dfrac{5e^2}{2}}\)

6216÷21 please show me the work

Answers

Therefore, on dividing 6216 by 21 we get 126

Given 6216 / 21

21 ) 6216 ( 296

      42

(-)...................     (By subtracting these two numbers we get )

      201            

       189

(-)..................    

          126

          126

.(-)..........................

            0

Therefore 6216 / 21 = 126

To learn more about divisions refer below

https://brainly.com/question/21416852

#SPJ9

A package contains 20 golf balls. The total
weight of the golf balls is 32 ounces. Use the
model to show the number of ounces per
golf ball and the number of golf balls per
ounce. Write your answers in the blanks.

Answers

Answer: 1.6

Step-by-step explanation:

each golf is 1.6 ounces and you 20 golf balls in order to reach 32 ounces

Answer:

shown below

Step-by-step explanation:

golf balls per ounce = 5/8 or 0.625

ounces per gold ball = 8/5 or 1.6

The midpoint of CD is M=(1, 2). One endpoint is C = (6, -2).
Find the coordinates of the other endpoint, D.

The midpoint of CD is M=(1, 2). One endpoint is C = (6, -2).Find the coordinates of the other endpoint,

Answers

Answer: Endpoint = (-4,6)

Step-by-step explanation:

To find any endpoint use the formula \(\frac{x_1+x_2}{2} = Xmp\)    \(\frac{y_1+y_2}{2} =Ymp\)

\(\frac{6+x_2}{2} = 1\)          \(\frac{-2+y_2}{2} =2\)    

multiply each by 2  to simplify

\(\frac{6+x_2}{2*2} = 1*2 = {6+x_2}= 2\)    \(\frac{-2+y_2}{2*2} =2*2 = -2+y_2=4\) take the equations and simplify

6+\(x_{2}\) = 2                          -2+\(y_{2}\) = 4  =  

-6       -6                          +2       +2

   \(x_{2}\) = -4                               \(y_{2}\) =  6

To check use the Midpoint formula

\(\frac{-4+6}{2} , \frac{6-2}{2} = \frac{2}{2} , \frac{4}{2} = (1,2)\)

You are interested in estimating the the mean age of the citizens living in your community. In order to do this, you plan on constructing a confidence interval; however, you are not sure how many citizens should be included in the sample. If you want your sample estimate to be within 5 years of the actual mean with a confidence level of 94%, how many citizens should be included in your sample? Assume that the standard deviation of the ages of all the citizens in this community is 22 years.

Answers

Answer:

The sample size is    \(n = 68 \)

Step-by-step explanation:

From the question we are told that

    The margin of error is  \(E = 5 \ years\)

     The standard deviation is  \(\sigma = 22\)

From the question we are told the confidence level is  94% , hence the level of significance is    

      \(\alpha = (100 - 94 ) \%\)

=>   \(\alpha = 0.06\)

Generally from the normal distribution table the critical value  of  \(\frac{\alpha }{2}\) is  

   \(Z_{\frac{\alpha }{2} } =  1.881\)

Generally the sample size is mathematically represented as  

   \(n = [\frac{Z_{\frac{\alpha }{2} } *  \sigma }{E} ] ^2\)

=>  \(n = [\frac{1.881 } *  22 }{5} ] ^2\)

=>  \(n = 68 \)

I'LL GIVE BRAINYEST

A box shaped like a rectangular prism has a height of 8 inches, a width of 6 inches, and a volume of 240 cubic inches. What is the length?
7 inches
6 inches
5 inches
4 inches

Answers

Answer:

7 inches..............

Answer:

8

Step-by-step explanation:

Volume = Length * Width * Height

512 = 8*8*Height

Height = 512/64=8

i just need the bottom one​

i just need the bottom one

Answers

The bottom of what?! I don't see anything

Answer:

X _> 3

Step-by-step explanation:

6/2 = 2x/2

3 = x

x = 3

A multiplication table. In the row labeled 3, the numbers 9, 12, 15, 18, and 21 are highlighted. In the row labeled 6, the numbers 18, 24, 30, 36, and 42 are highlighted. Look at the highlighted portions in the rows for 3 and 6. What do the two sets of numbers have in common? They both show products of multiplying by

Answers

The common number in both sets will be 18. And the row labeled 6 is the multiplication of the row labeled 3 with 2.

What is Algebra?

The analysis of mathematical representations is algebra, and the handling of those symbols is logic.

A multiplication table.

In the row labeled 3, the numbers 9, 12, 15, 18, and 21 are highlighted.

In the row labeled 6, the numbers 18, 24, 30, 36, and 42 are highlighted.

Look at the highlighted portions in rows 3 and 6.

The common number in both sets will be 18.

The row labeled 6 is the multiplication of the row labeled 3 with 2.

More about the Algebra link is given below.

https://brainly.com/question/953809

#SPJ1

Answer: the person above is wrong

its 3, 4, 5, 6, and 7

What does the enlargement look like and what are the coordinates?

What does the enlargement look like and what are the coordinates?

Answers

Answer:

(-6, -3) (-3,-3) (-3,-6)

Step-by-step explanation:

Multiple all of the vertices by 1.5.

(-6, -3) (-3,-3) (-3,-6)

Hope I helped!

[enlargment will still be a triangle]

Consider the first quadrant of the unit circle. How does the covenant ratio change as the sine ratio increases?

Answers

Answer:

For acute angles, remember what sine means: opposite over hypotenuse. If we increase the angle, then the opposite side gets larger. That means "opposite/hypotenuse" gets larger or increases.

Step by Step:

Keep this in mind >>

Consider the unit circle > The sine and cosine ratios are the only ratios that have 1 (the radius or hypotenuse) as the denominator. The numerators (sides) vary between 0 and 1, thus determining that the sine and cosine do the same.

All of the other ratios (tangent, cotangent, secant, cosecant) have a side as the denominator, varying between 0 and 1. As any denominator approaches 0, the value of the ratio approaches infinity.

Is the product of 3/6 times 5/4 equal to the the product of 3/4 times 5/6? Explain

Answers

Answer:

Step-by-step explanation:

Yes the product is equal because even though the numbers may have changed places they did not change the position(numerator to denominator) vice versa.meaning the product of what is in the numerator is equal irrespective of the positions for an e.g in numerator part ion the first terms we have 3*5=15 and in the second scenario we have of course 3*5=15 same applies to the denominators 3*5=5*3

Find the distance between the points (6, -4) and (-7, -4)

Answers

Answer:

8

Step-by-step explanation:

Subtract 6 to 7 then add that to the 7.

Please help me i need help!! How do you solve for M? Explain each step of the solution in a short paragraph

Please help me i need help!! How do you solve for M? Explain each step of the solution in a short paragraph

Answers

Answer:

M = 4pi²a³ / GT²

Step-by-step explanation:

Cross multiply: T² × GM = GMT²

1 × 4pi²a³ = 4pi²a³

It becomes: GMT² = 4pi²a³

Divide both sides by GT² to solve for M

We have : M = 4pi²a³ / GT²

Find the measure of the requested angle.

d =

e =

Find the measure of the requested angle. d = e =

Answers

Answer:

e is 63 degrees

d is 31 degrees

In the first quadrant you start at (7,4) and move 4 units left and 3 units up.

Answers

Answer:

(3,7)

Step-by-step explanation:

(7,4)

Moving left and right relates to x

Moving up and down relates to y

When you move to the left, you subtract

When you move to the right, you add

When moving up, you add

When moving down you subtract

X= 7-4 = 3

x=3

Y= 4+3 = 7

y = 7

(3,7)

The theater sells two types of tickets: adult tickets for $6 and child tickets for $4.
Last night, the theater sold a total of 364 tickets for a total of $1930. How many adult tickets did the theater sell last night?

Answers

Answer:

237

Step-by-step explanation:

This is a system of equations.

The theater sold 364 adult and child tickets, so a + c = 364

They made a total of $1930. Each adult ticket was $6 & child tickets were $4. The second equation is 6a + 4c = 1930.

Let's line them up

a  +  c = 364

6a + 4c = 1930

Since we need to solve for the number of adult tickets, we want to get rid of the c variable. I'm going to multiply the entire first equation by -4 to do this. The second equation stays the same. Now, I have:

-4a - 4c = -1456

6a + 4c = 1930            Add them together

----------------------

2a = 474                      Divide by 2 to solve for a

a = 237

There were 237 adult tickets sold

What is the value of v? (help no links and thx)

What is the value of v? (help no links and thx)

Answers

Answer:

74.7

Step-by-step explanation:

HELP
Recipe:
Cinnamom Rolls
makes 12 rolls
1/4 -ounce package active dry yeasy
1 cupt warm water
3 tablespoons molasses
1/3 cup powedered milk
1 egg yolk,beaten
1 egg white, beaten
3 1/2 cups bread flour, divided
6 tablespoons butter, melted and divided
1 teaspoon salt
1/2 cup whole wheat flour
1/3 cup raisins
1 large egg, beaten
​​​​​​​1/4 cup water

You will use the recipe above to answer the following questions:

Respond to the questions about the second recipe:
This recipe makes 12 rolls, but you need to make 30 rolls. What number will you need to multiply the amount of each ingredient by to adjust the recipe? How did you determine this number?
How many ounces of yeast will you need to make 30 rolls?
How many cups of powdered milk will you need to make 30 rolls?
How many tablespoons of butter will you need to make 30 rolls?
How many more cups of bread flour than whole wheat flour will you need to make 30 rolls?
Notice that the original recipe calls for
1
4
cup of water. If you put in
3
8
cup of water, by how much have you

Answers

To adjust the recipe to make 30 rolls instead of 12, we need to multiply the amount of each ingredient by a factor of 30/12 = 2.5.

What is factor?

A factor is a number or expression that divides another number or expression evenly without a remainder.

According to question:

To adjust the recipe to make 30 rolls instead of 12, we need to multiply the amount of each ingredient by a factor of 30/12 = 2.5.

To determine this factor, we divide the desired number of rolls (30) by the original number of rolls (12):

2.5 = 30/12

To find the amounts of each ingredient needed to make 30 rolls, we multiply the original amounts by 2.5:

Yeast: 1/4 oz x 2.5 = 0.625 oz (or about 0.63 oz)

Warm water: 1 cup x 2.5 = 2.5 cups

Molasses: 3 tbsp x 2.5 = 7.5 tbsp (or about 1/2 cup + 1 tbsp)

Powdered milk: 1/3 cup x 2.5 = 0.83 cups (or about 3/4 cup + 1 tbsp)

Egg yolk: 1 beaten yolk x 2.5 = 2.5 beaten yolks (or about 2 yolks)

Egg white: 1 beaten white x 2.5 = 2.5 beaten whites (or about 2 whites)

Bread flour: 3 1/2 cups x 2.5 = 8.75 cups (or about 8 3/4 cups)

Butter: 6 tbsp x 2.5 = 15 tbsp (or about 1 cup - 1 tbsp)

Salt: 1 tsp x 2.5 = 2.5 tsp (or about 2 1/2 tsp)

Whole wheat flour: 1/2 cup x 2.5 = 1.25 cups (or about 1 1/4 cups)

Raisins: 1/3 cup x 2.5 = 0.83 cups (or about 3/4 cup + 1 tbsp)

To make 30 rolls, you will need 0.625 oz (or about 0.63 oz) of yeast.

To make 30 rolls, you will need 0.83 cups (or about 3/4 cup + 1 tbsp) of powdered milk.

To make 30 rolls, you will need 15 tbsp (or about 1 cup - 1 tbsp) of butter.

To make 30 rolls, you will need 8.75 cups (or about 8 3/4 cups) of bread flour, which is 8.75 - 1.25 = 7.5 cups more than the 1.25 cups of whole wheat flour needed.

If you put in 3/8 cup of water instead of the 1/4 cup called for in the original recipe, you have added an extra 3/8 - 1/4 = 1/8 cup of water, or 2 tablespoons of water.

To know more about factor visit:

https://brainly.com/question/29014674

#SPJ1

Solve for w.

1/3w + 5/6w - 2 = -w

Answers

Answer: \(w=\frac{12}{13}\)

Step-by-step explanation:

To solve for w, we want to isolate the variable.

\(\frac{1}{3}w +\frac{5}{6}w-2=-w\)        [add both sides by 2 and w]

\(\frac{1}{3}w +\frac{5}{6}w+w=2\)           [convert to same denominator]

\(\frac{2}{6}w +\frac{5}{6}w+\frac{6}{6} w=2\)         [add]

\(\frac{13}{6} w=2\)                          [multiply both sides by 6/13]

\(w=\frac{12}{13}\)

Now we know that \(w=\frac{12}{13}\).

Answer:

Step-by-step explanation:

1/3w + 5/6w - 2 = -w    Collect like terms on the left.

Before I do, I'm going to assume the question is (1/3)w + (5/6)w - 2 = - w

(1/3)w + (5/6)w - 2 = - w

 

1/3 + 5/6 = 2/6 + 5/6 = 7/6

(7/6)w - 2 = -w             Add w to both sides

(7/6)w + w - 2 = 0        Combine the left

(7/6)w + 6/6 w - 2 = 0

(13 / 6) w - 2 = 0          Add 2 to both sides

(13/ 6) w = 2                 Divide by 13/6

w = (2/1 ) ÷ (13/6)          Invert the denominator and multiply

w = 2/1 * 6/13

w = 12/13    

             


Find the accumulated value of an investment of $20,000 for 7 years at an interest rate of 4% if the money is a.
compounded semiannually; b. compounded quarterly; c. compounded monthly; d. compounded continuously.

Answers

Part a; compounded semiannually (n = 2); CI =  $6,390.

Part b: compounded quarterly ( n = 4) ; CI =  $6,425

Part c: compounded monthly (n = 12): CI =  $6,450

Part d: compounded continuously; CI =  $6,467.

What is known as the term compound interest?Compound interest is computed as interest on the base principal plus any interest income from prior periods. The frequency plan for compounding interest can be set to be continuous, daily, or yearly.

The compound interest is calumniated by formula.

A = P(1 + r/100n)∧nt

In which,

A  = amount after compounding

P = principal amount  ($20,000 )

n = number of time principal amount compounded in a year.

t = total time in years (7 years)

r= rate of interest (4%)

Part a; compounded semiannually (n = 2)

A = P(1 + r/100n)∧nt

A = 20,000(1 + 4/200)∧(2x7)

A = 26390

CI = A - P

CI =  26,390 -  20,000

CI =  $6,390.

Part b: compounded quarterly ( n = 4)

A = 20,000(1 + 4/400)∧(4x7)

A = 26425

CI = A - P

CI =  26,425 -  20,000

CI =  $6,425

Part c: compounded monthly (n = 12)

A = 20,000(1 + 4/1200)∧(12x7)

A = 26450

CI = A - P

CI =  26,450-  20,000

CI =  $6,450

Part d: compounded continuously.

P = P₀e∧rt

P = (20,000)(2.72)∧(0.04 x 7)

P = 20,000 x 1.32

P = 26,467

CI =  26,467 - 20, 000

CI =  $6,467.

Thus, the for compounded continuously is $6,467.

To know more about the compound interest, here

https://brainly.com/question/28020457

#SPJ1

Other Questions
1. (poder) Poderias t ayudarme con esta tarea? (imperative command)2. Yo en t lugar (ponerse) el vestido azul. (present tense)3. T (tener) _________ tiempo de venir estar noche? (preterite tense)4. En otra poca esa casa (valer) ________ mucho menos. (imperfect tense)5. A mi fiesta (venir) _ ______ms gente que a la tuya. (future tense)6. Los estudiantes __________ regalan esos libros de cuentos. (to us)/ (double object pronoun)7. Yo creo que tu hermana menor (saber) ____ ______ms de historia que t. (present tense)8. Qu le (decir) ___________el jefe a Corina? (preterite tense)9. (hacer) __________ Vd. ms pasteles. Recuerda que son 10 nios invitados. (imperative tense)10. Jos (querer) __ _______ hacer lo mismo que hace su hermano. (future tense)11. La nia quiere comprar____________. (for her parents) (the picture) (double object pronoun)12. Lo hice __________ mi familia. (for)13. El beb van a comer___. (the apples) DOP.14. (Poner) _____ Vds. (the table) hoy. (DOP) la mesa15. Tus abuelos _________ (acostarse) temprano los das entre semana. (present tense) 1. What sense does Krueger think is the most dominant?2. Do you think the perceptions that people have while in an artificial reality environment are real? (this is psychology by the way. Help) Helps distribute chromosomes to new cells during cell division. WIll mark brainliest on january 1, 2019, miller corporation had retained earnings of $8,100,000. during 2019, miller reported net income of $1,400,000, declared dividends of $400,000, and issued common stock for $1,110,000. what were miller's retained earnings on december 31, 2019? Which of the following is an equation to the tangent line to the graph of (fx) = -2x+4 at 1A) y= -2(x-1) + 2B) y= 2(x-1) + 2C) y= -2(x+1) + 2D) y= 2(x-1) - 2Please show how to find this as I really want to learn! A model for the product of two numbers is shown below. Each large rectangle represents one whole.UUUUUWhich expression does this model represent?OA. 0.6 x 7.5OB. 60 x 7.5OC. 0.6 0.75OD 6.0 x 0.75ResetSubmit2 of 10 Answered If m = 3,000, p = 2, and y = 12,000, what is velocity? What claim put forward by those opposed to a strong and independent judiciary does hamilton address?. Scientists are interested in determining if selenium, from a nearby mine, magnifies in the tissues of fish living in a lake Which of the following best describes a testable hypothesis for the study? Tissue from fish in the lake will have a higher level of selenium than tissue from fish in a nearby take O Fish will have a higher concentration of selenium than mercury in their tissues O Selenium will be at a higher concentration in fish tissue than in the tissue of fish prey. O Selenium will be at a higher concentration in acid mine drainage than in fish tissues solve for g 2(g - 4) = 6 what is the simplified product of 2/9 x 3/8A. 6/72. B. 5/17. C. 3/36. D.1/12 PLZ ANSWER THE QUESTION Helpppp!!! The question says which pair of linear equations described the graph of two perpendicular lines? The options areA. Y-2=3(x+1) Y+4=1/3(x-5)B. Y-2=3(x+1) Y+4=3(x-5)C. Y-2=3(x+1) Y+4=-1/3(x-5)D. Y-2=3(x+1) Y+4=-3(x-5) _________ were radical reformers in Germany and Switzerland who emphasized that baptism should only be of adults and that Christians should separate themselves into communities of the "truly redeemed." the client presents to the clinic with complaints of a painful rash under the left breast. the nurse observes a red papular rash and suspects the client is suffering from: Find the sale price with percent markdown A store advertises a 35% markdown on the PlayStation 5 gaming console. It normally sells for $1,019. How much would you save on the console? And what is the final sale price? 7/20 + ___=19/20 is the answer 3/5 Because of all of their adventures, what class did Summer say they would be ahead in?GeometryGeographyHistorySocial Studies In Original Source 2, "Development and initial validation of the Hangover Symptoms Scale: Prevalence and correlates of hangover symptoms in college students," one of the questions was how many times respondents had experienced at least one hangover symptom in the past year. Table 3 of the paper (page 1,445) shows that out of all 1,216 respondents, 40% answered that it was two or fewer times. Suppose the researchers are interested in the proportion of the population who would answer that it was two or fewer times. Can they conclude that this population proportion is significantly less than half (50% or 0.5)? Go through the four steps of hypothesis testing for this situation for women only. Use appropriate software or a calculator to find the standard deviation and the test statistic. (Round your standard deviation to three decimal places and your test statistic to two decimal places.) Test Statistic?